please help! asap! thanks! xoxoxo

Please Help! Asap! Thanks! Xoxoxo

Answers

Answer 1

Answer:

B

Step-by-step explanation:

Don't know how to type those here so see attachment

Please Help! Asap! Thanks! Xoxoxo
Answer 2

Answer:

x = √29

Step-by-step explanation:

a^2 + b^2 = c^2

5^2 + 2^2 = c^2

25 + 4 = c^2

29 = c^2

√29 = √c^2

√29 = c

x = √29


Related Questions

Justin and his children went into a grocery store and he bought $18 worth of bananas and mangos. Each banana costs $0.60 and each mango costs $1.50. He bought 2 more bananas than mangos. Graphically solve a system of equations to determine the number of bananas, x, and the number of mangos, y, that Justin bought.

Write an answer in y=mx+b form, please.
Two equations.

Answers

Answer:

x=10 and y= 8.

Step-by-step explanation:

Given that the cost of 1 banana = $0.60

Cost of 1 mango = $1.50.

Total cost= $ 18  

Here, x be the number of bananas and y be the numbers of mangos,

As the number of bananas is 2 more than the number of mangos,

So, x=y+2

[tex]\Rightarrow y=x-2 \cdots(i)[/tex]

Cost of x bananas [tex]=\$ 0.60 \times x[/tex]

Cost of y mangos [tex]=\$ 1.50 \times y[/tex]

Total cost = 0.6x+1.5y

[tex]\Rightarrow 18 = 0.6x+1.5y\\\\\Rightarrow 1.5y=18-0.6x\\\\\Rightarrow y= 12 - 0.4 x \cdots(ii).[/tex]

Solving both the equations (i) as well as (ii), graphically as shown in the figure by plotting both the graphs.

Both the graphs intersect at the point (10,8) as shown in the graph.

So, the solution of both the equations is, (x,y)=(10,8). i.e

The number of bananas = x= 10 and

the number of mangos = y = 8.

what expression is equal to(5+x-2)×2

Answers

If you simply the equation it would be 6+2x

Brad had 10 crickets in his pet lizard’s cage. After one day, Brad’s lizard has eaten 20% of the crickets he had put in the cage. By the end of the next day, the lizard had eaten 25% of the remaining crickets. how many crickets were left in the cage at the end of the second day?

Answers

7.5 crickets left (I think)

20%=2

25%(of 2)=0.5

2+.5=2.5

10-2.5=7.5

Hope this helps:)

A bag contains 95 blue marbles, 7 red marbles, and 5 green marbles. One marble is randomly
chosen. Select all true statements about the marble chosen.
A It is certain that the marble was blue.
B. It is unlikely that the marble chosen was green.
C. It is unlikely that the marble chosen was orange.
D. It is likely that the marble chosen was blue.
E It is not possible that the marble chosen was green.
E It is neither likely nor unlikely that the marble chosen was red.

Answers

Answer:

B,D, and the last E

Step-by-step explanation:

B,d,f is that good for you

A cab company charges $3.10 flat rate in addition to $0.85 per mile. Rex has no more than $15 to spend on a ride. Write an inequality that represents Rex's situation. How many miles can Rex travel without exceeding his limit? Round off your answer to nearest tenth.

Answers

Answer:

Step-by-step explanation:

Let the total number of miles Rex can travel be x;

If a cab company charges $0.85 per mile, then x miles will cost $0.85x

If the flat rate rate charge is $3.10, the total price for x miles will be;

0.85x + 3.10

Since Rex has no more than $15 to spend on a ride, the inequality to represent the equation will be;

0.85x + 3.10 ≤ 15 (less than or equal to means that the total value cannot exceed $15)

Next is to solve for x

Given

0.85x + 3.10 ≤ 15

subtract 3.10 from both sides

0.85x + 3.10-3.10 ≤ 15-3.10

0.85x ≤ 15-3.10

0.85x ≤ 11.90

x ≤  11.90/0.85

x ≤ 14

This means that Rex can travel 14 miles without exceeding his limit

Answer:

[tex]3.10 + 0.85m \leq 15[/tex]

[tex]m \leq 14.0[/tex]

Step-by-step explanation:

Given

[tex]Flat\ Rate = \$3.10[/tex]

[tex]Addition = \$0.85[/tex] (per mile)

[tex]Maximum\ Amount = \$15[/tex]

Required

Determine the inequality that represents the scenario and solve

Let the number of miles be represented by m.

The company's charges can be calculated using:

[tex]Flat\ Rate + Additional\ Charges * m[/tex]

Substitute values

[tex]\$3.10 + \$0.85 * m[/tex]

Rex can't exceed $15 implies that the company's charges can't exceed Rex's budget.

This is expressed as:

[tex]\$3.10 + \$0.85 * m \leq \$15[/tex]

[tex]\$3.10 + \$0.85m \leq \$15[/tex]

[tex]3.10 + 0.85m \leq 15[/tex] ---- The inequality

Solving for m: Collect Like Terms

[tex]0.85m \leq 15 - 3.10[/tex]

[tex]0.85m \leq 11.9[/tex]

Divide through by 0.85

[tex]m \leq 11.9/0.85[/tex]

[tex]m \leq 14.0[/tex] ---- The solution

A retail grocer bought a case of 12 packages of coffee for $52.32.

How much did the retailer pay for each package of coffee?


The retailer sold each package for $12.59.

What is the difference between the retailer’s cost and the selling price for each package of coffee?

Answers

Answer:

4.36

8.23

Step-by-step explanation:

Answer:

4.36

Step-by-step explanation:

I need help please ​

Answers

(-5,1) is the correct answer or option c:)

44.8 is what percent of 160?

Answers

Answer:

28% .................

Answer:

28%

Step-by-step explanation:

100%/x%=160/44.8

(100/x)*x=(160/44.8)*x       - we multiply both sides of the equation by x

100=3.5714285714286*x       - we divide both sides of the equation by (3.5714285714286) to get x

100/3.5714285714286=x

28=x

x=28

now we have:

44.8 is 28% of 160

Given the following linear function sketch the graph of the function and find the domain and range. ƒ(x) = -5x + 4

Answers

Answer:

Domain: (-∞,∞)

Range:(-∞,∞)

Step-by-step explanation:

The sketch of the given function has been attached and the domain and range both will be (-∞,∞ ).

What is a linear function?

A straight line on the coordinate plane is represented by a linear function.

A function that fluctuates linearly with respect to the changing variable is referred to as being linear.

A linear function always has the same and constant slope.

The formula for a linear function is f(x) = ax + b, where a and b are real values.

Given the function,

ƒ(x) = -5x + 4

Range of the function ;

Since the value of the function is defined for all real x and the value will be unique so

Range =  (-∞,∞ ).

The domain of the function ;

Since it is defined for all real x so,

Domain =  (-∞,∞ )

For more about the linear function

brainly.com/question/21107621,

#SPJ2

The figures below are similar. Find the length of the missing side.
90
10
7
Х
36
4
14
126
X=

Answers

Answer:

63

Step-by-step explanation:

multiply the side by 9

Solving the system using equations ( 15 points)
3x+2y=17
2x+5y=8

Answers

Answer:

{x,y}={  11 /69   ,−  11 /10  }

Step-by-step explanation:

Choose the correct answer choice.

Answers

Answer:

The T shirt choice

Step-by-step explanation:

Both values increase at the same rate therefore is proportional.

simplify (-sqrt of 125)^3

Answers

Answer:

5

Step-by-step explanation:

[tex]\sqrt[3]{125} =5*5*5=5[/tex]

Hellllllllllllppppppppppp

Answers

Answer:

The answer is 5 1/2! So what you are looking for is 1/2.

Step-by-step explanation:

Answer:

11/2

Step-by-step explanation:

Can somebody plz answer all of them correct! (Only if u done this before)
thanks! WILL MARK BRAINLIEST
(Don’t judge me..I didn’t study so I need to do corrections)

Answers

Answer:

23) 53/100

24)2/5

25)3/5

26)11/50

27)17/50

28)19/1000

29)4/5

30)1/250

31)9/25

32)1 3/10

33)11 1/2

34) 7 3/40

Step-by-step explanation: Hope this helps!

IL GIVE BRAINLIST!! PLEASE HELP! THERES 2 ANSWERS JUST SAYING!!​

Answers

Answer: Option 2

Pls mark me brainilest!

Step-by-step explanation:

I don’t understand.

Answers

Answer:

y = -2/3x

Step-by-step explanation:

A perpendicular line is the negative reverse of the slope of the other line. I'm not sure about the options you have there but hopefully this helps you get your answer.

Solve the triangle, find m∠A and m∠C. Round angles to the nearest degree.
m∠A= __∘

m∠C= __∘

Answers

Answer:

[tex]m\angle A=63^\circ\\m\angle C=26^\circ[/tex]

Step-by-step explanation:

Trigonometric Ratios

The ratios of the sides of a right triangle are called trigonometric ratios. The longest side of the triangle is called the hypotenuse and the other two sides are called the legs.

Selecting any of the acute angles as a reference, it has an adjacent side and an opposite side. The trigonometric ratios are defined upon those sides.

The cosine ratio is defined as:

[tex]\displaystyle \cos\theta=\frac{\text{adjacent leg}}{\text{hypotenuse}}[/tex]

Note the angle A of the figure has 17 as the adjacent leg and 38 as the hypotenuse, so we can directly apply the formula:

[tex]\displaystyle \cos A=\frac{17}{38}[/tex]

[tex]\cos A=0.4474[/tex]

Using a scientific calculator, we get the inverse cosine:

[tex]A=\arccos(0.4474)[/tex]

[tex]A\approx 63^\circ[/tex]

Since A+B+C=180°, we can solve for C:

C = 180° - A - B

C = 180° - 63° - 90°

C = 26°

Thus:

[tex]m\angle A=63^\circ\\m\angle C=26^\circ[/tex]

In the given right triangle ABC, m∠A ≈ 26.44° and m∠C ≈ 63.56°.

To solve the right triangle ABC, we can use trigonometric ratios. In a right triangle, the three main trigonometric ratios are:

1. Sine (sin): [tex]\(\sin(\theta) = \frac{{\text{opposite side}}}{{\text{hypotenuse}}}\)[/tex]

2. Cosine (cos): [tex]\(\cos(\theta) = \frac{{\text{adjacent side}}}{{\text{hypotenuse}}}\)[/tex]

3. Tangent (tan): [tex]\(\tan(\theta) = \frac{{\text{opposite side}}}{{\text{adjacent side}}}\)[/tex]

Given:

AC = 38

AB = 17

To find the angles m∠A and m∠C, we can use the sine and cosine ratios, respectively.

1. For m∠A:

[tex]\(\sin(m\angle A) = \frac{{AB}}{{AC}} = \frac{{17}}{{38}}\)\\\\\(m\angle A= \sin^{-1}\left(\frac{{17}}{{38}}\right)\)[/tex]

2. For m∠C:

[tex]\(\cos(m\angle C) = \frac{{AB}}{{AC}} = \frac{{17}}{{38}}\)\\\\\(m\angle C = \cos^{-1}\left(\frac{{17}}{{38}}\right)\)[/tex]

Let's calculate the angles:

[tex]1. \(m\angle A \approx \sin^{-1}\left(\frac{{17}}{{38}}\right) \approx 26.44^\circ\)\\\\2. \(m\angle C \approx \cos^{-1}\left(\frac{{17}}{{38}}\right) \approx 63.56^\circ\)[/tex]

Therefore, m∠A ≈ 26.44° and m∠C ≈ 63.56° (rounded to the nearest degree).

To know more about right triangle, refer here:

https://brainly.com/question/31613708

#SPJ2

Same factory has supplies expenses of $1135.78 a day, in addition to the wages above and makes $60,128.72 in one week, what is the factory's total profit or loss in one week?

Answers

Given :

Factory has supplies expenses of $1135.78 a day.

Company earns $60,128.72 in one week.

To Find :

The factory's total profit or loss in one week.

Solution :

Expenses of factory of a week :

E =$( 1135.78 × 7 ) = $7950.46 .

Now, profit is given by :

P = $60,128.72 - $7950.46

P = $52178.26

Therefore, total profit of factory in one week is $52178.26 .

Hence, this is the required solution.

What is the image point of (-3,3) after the transformation R180° 0 T-3, -4?

Answers

Answer:

Image point(6, 1)

Step-by-step explanation:

Given point → (-3, 3)

Transformation to be done → [tex]R_{180}0T_{-3,-4}[/tex]

Transformations to be done,

Step - (1). Translation of the given by 3 units left and 4 units down.

Step - (2). Followed by the rotation counterclockwise 180° about the origin.

Rule for step (1),

(x, y) → (x - 3, y - 4)

By this rule,

(-3, 3) → [(-3 - 3), (3 - 4)]

         → (-6, -1)

Rule for step -2,

(x, y) → (-x, -y)

(-6, -1) → (6, 1)

Therefore, following these two steps coordinates of the image point → (6, 1)  

One Saturday, a butcher sells a total of b pounds of beef at $6.00 per pound. She also sells some pork for $7.00. On Sunday she sells b pounds of beef again, but at the sale price of $4.00 per pound. She also sells some pork for $9.00. Given that she made the same revenue of d dollars each day, which of the following systems of equations can be used to find out how many pounds of beef, b, she sold each day?

Answers

The equations that can be used to find out how many pounds of beef, b, she sold each day will be:

d = 6b + 7.

d = 4b + 9

How to illustrate the equation?

From the information given, the butcher sells a total of b pounds of beef at $6.00 per pound and she also sells some pork for $7.00.

Therefore, the equation will be:

d = (6 × b) + 7.

d = 6b + 7.

d = (4 × b) + 9

d = 4b + 9

Learn more about equations on:

brainly.com/question/13448052

#SPJ1

A newspaper is rolled into a cylindrical shape of approximate diameter 4cm It is wrapped for posting with a strip of paper which goes about 2 1/2 times round the newspaper Use the value 3 for π to find the approximate length of the wrapping paper. And 2. Calculate the total surface area of a solid cone of slant hight 10cm and base diameter 10 cm.Use the value 3.14 for π

Answers

Answer:

30 cm

[tex]863.5\ \text{cm}^2[/tex]

Step-by-step explanation:

d = Diameter of cylinder = 4 cm

n = Number of times the strip of paper is turned = [tex]2\dfrac{1}{2}=2.5[/tex]

Diameter of the cylinder will be approximately equal to the diameter of the paper wound. Length of one turn will be circumference of the paper

Circumference of the paper

[tex]c=\pi d\\\Rightarrow c=3\times 4\\\Rightarrow c=12\ \text{cm}[/tex]

Total length of the paper will be the number of turns multiplied by the length of one turn

[tex]l=nc\\\Rightarrow l=2.5\times 12\\\Rightarrow l=30\ \text{cm}[/tex]

Length of the strip of paper is 30 cm.

2.

l = Slant height = 10 cm

d = Base diameter = 10 cm

r = Radius of base = [tex]r=\dfrac{d}{2}=\dfrac{10}{2}=5\ \text{cm}[/tex]

Total surface area of cone is given by

[tex]S=\pi r^2+\pi rl\\\Rightarrow S=\pi r(r+rl)\\\Rightarrow S=3.14\times 5(5+5\times10)\\\Rightarrow S=863.5\ \text{cm}^2[/tex]

The total surface area of cone is [tex]863.5\ \text{cm}^2[/tex].

An isosceles triangle has base angles that each measure 42 degrees.Which equation can be used to find z, the measure of the third angle of this isosceles triangle in degrees

Answers

Answer:

z+42+42=180

Step-by-step explanation:

Since it is an isosceles triangle, the two base angles are the same and remember all the angles add to  180. So use this equation:

z+42+42=180

If you need to find z, it is 96.

Check: 96+42+42=180

Henry left for the lake in the morning and it was 78°F.
When he arrived at the lake the temperature had increased
7° and by lunch it increased another 5°. What is the
temperature when Henry was eating lunch?

Answers

Answer:

The temputure is -64

Step-by-step explanation:

QUICK!!!!!

Analyze the conditional statement below and complete the instructions that follow.

If m is the midpoint of AB, then M divides AB into two congruent segments.

Identify the inverse of the converse of the conditional statement.

O If M is not the midpoint of AB, then M does not divide AB into two congruent segments.

If M is the midpoint of AB, then M divides AB into two congruent segments.

If M divides AB into two congruent segments, then M is the midpoint of AB.

Olf M does not divide AB into two congruent segments, then M is not the midpoint of AB.

Answers

Answer:

D. If M does not divide AB into two congruent segments, then M is not the midpoint of AB.

Step-by-step explanation:

A conditional statement is one that include 'if'. Thus it is also referred to as an 'if' statement.

Given a conditional statement:

If M is the midpoint of AB, then M divides AB into two congruent segments.

The converse of the given statement is done by interchanging the two parts of it. So that we have:

If M divides AB into two congruent segments, then M is the midpoint of AB.

Then, the inverse can be obtained by getting the negative of both parts of the converse. Therefore, the inverse is:

If M does not divide AB into two congruent segments, then M is not the midpoint of AB.

The correct option is D.

what does angle DFE= in degrees?
Type only number for answer​

Answers

answer: 40°

explanation: angle AFB and DFE are vertical angles meaning they share the same measurements

Can someone help me cuz i need help

Answers

Answer:

f

klllllllllk

Step-by-step explanation:

C and E are true
———————————

if g(x)=x²-2 and p(x)=2x+3, find (g·p)(x)

Answers

2X^3+3x^2-4x-6

You do x^2-2 times 2x+3 and use FOIL method to get answer

3*10^4+2.5*10^0
ghfuigffhh

Answers

Answer:

30002.5

Step-by-step explanation:

YALL BETTER ANSWER THIS PLSSSSSSSSSSS

Answers

Answer:

= 3/16(0.1875)

Step-by-step explanation:

explanation in the image.

Answer:

Step-by-step explanation:

Take out lignes

-1/4x-3/7

= -7/12

Other Questions
3s - 4 = 8 cant figure it out 40 mangoes cost $16, if the rate remains the same, how many mangoes can be purchased with $4 An angry mob of over 1,000 whites gathers in front of Central HighSchool, while nine African American students are escorted inside. TheLittle Rock police remove the nine children for their safety.-Little Rock Central High Schoo l National Historic Site, NationalPark Service, www.nps.gov (accessed December 2, 2013)This confrontation centered around- A.disagreement over the use of taxes to fund public educationB. the refusal of federal courts to hear cases concerning civil rights violationsC. the denial of First and Fifth Amendment freedoms by southern state legislaturesD. resistance by state and local governments to the Brown v. Board of Education ruling Find the slope of the line that passes through the points (10,8) and (4,12). WRITE THE ANSWER AS A SIMPLIFIED FRACTION PLEASE NEED HELP PLEASE!!!! Y=-5x+2 fill in the table using this function rule What is a pair of complex numbers called? here is a question if you want to answer Write a two page essay on Alexander Hamiltons life and his accomplishments . Why were advances in printing words so important in the evolution of graphic design?Printing words helped people learn how to print images.As printing technology evolved, it was possible to recreate images.Images are easier to understand than written language, so they evolved first.Words and images did not appear together before the invention of the printing press Question 9 of 10The album Thriller was released in 1982 by Michael Jackson and was thelargest selling album of all time. Name the producer.A. Ray CharlesB. Quincy JonesC. Phil SpectorD. PrinceSINMIT Which of the following transactions are included in gross domestic product, and by how much does the GDP raise? A. Smith pays a carpenter $50,000 to build a garage. B. Smith purchases $10,000 worth of materials and builds a garage, which is worth $50,000. C. Smith goes to the woods, cut down a tree, and uses the wood to build himself a garage that is worth $50,000. D. The Jones family sells its old house to the Reynolds family for $400,000. The Joneses then buy a newly constructed house from a builder for $500,000. E. You purchase a used computer from a friend for $200. F. Your university purchases a new mainframe computer from IBM, paying $25,000. G. You win $100 in an Atlantic City casino. H. You make $100 in the stock market. I. You sell a used economics textbook to your college bookstore for $60. J. You buy a new economics textbook from your college bookstore for $100. yoo someone help me with this im at school right now and its due in like 10 mins ! In sea-floor spreading, molten material rises from the mantle and erupts in the... is aluminum Pure or Impure? Insert commas where necessary in the sentence below:Our newest staff member Lamar Roberts graduated in May. Help me please lol!!!!!! A blankcolor is created by mixing together a primary and a secondary color..coolOB. tertiaryO C.warmD.neutral your sister has gained admission into your former school. write a letter of about 450 word giving her information about the school and advise her on how to behave well while in the An object has a mass of 15 kg and is accelerating to the right at 16.3 m/s2. The free-body diagram shows the horizontal forces acting on the object. A free body diagram with 2 forces. The first vector is pointing right, labeled F Subscript a Baseline 250 N. The second vector is shorter pointing left, labeled F Subscript f Baseline. What is the frictional force, Ff, acting on the object?